LSAT and Law School Admissions Forum

Get expert LSAT preparation and law school admissions advice from PowerScore Test Preparation.

User avatar
 Dave Killoran
PowerScore Staff
  • PowerScore Staff
  • Posts: 5852
  • Joined: Mar 25, 2011
|
#49160
Complete Question Explanation
(The complete setup for this game can be found here: lsat/viewtopic.php?t=9142)

The correct answer choice is (E)

If O is prescribed, then from the second rule M and O cannot be prescribed. Accordingly, answer choices (A) and (C) can be eliminated. Also, from our discussion of inferences, we learned that O and V cannot be prescribed together. Consequently, answer choices (B) and (D) can be eliminated. Thus, answer choice (E) is correct.
 lp1997
  • Posts: 12
  • Joined: Apr 23, 2018
|
#46426
If O is prescribed for a given case, which one of the following is a pair of treatments both of which must also be prescribed for that case?

My initial inference was that M and N must automatically be excluded because of the 1 dietary restriction rule. (This eliminated answer choices A and C) However, I could not figure out anything else from there using any inferences, so I ended up creating several hypotheticals. Are there any inferences that I am missing that could have made the process quicker?
Thanks.
User avatar
 Jonathan Evans
PowerScore Staff
  • PowerScore Staff
  • Posts: 726
  • Joined: Jun 09, 2016
|
#46468
Hi, NSchlesi,

Good question. A lot of this game hinges on observing the interactions between the kinds of items selected and not selected, as you did here. Once you exclude M and N, let's see what else happens.
  • M :arrow: V
This rule (the contrapositive of the sixth rule) allows you to fill in the rest of the blanks.
  • Selection: O __ __ __ __ Not Selected: M N V __
Remember that we have exactly one Dietary Regimen and either one or two Antibiotics. If we have a maximum of two Antibiotics, we must have a minimum of two Physical Therapies:
  • Selection: D A A PT PT Not Selected: D D PT A
In this problem we know that one of the physical therapies is not prescribed. This means that the other two must be prescribed. Let's see where this takes us:
  • Selection: O U W __ __ Not Selected: M N V __
Once we know that both U and W are prescribed, what else do we know?
  • W :arrow: F
We know F is not prescribed:
  • Selection: O U W __ __ Not Selected: M N V F
Therefore the other two Antibiotics must be prescribed:
  • Selection: O U W G H Not Selected: M N V F
This allows us to get the correct answer, (E), U and W.
 nschlesi
  • Posts: 6
  • Joined: May 25, 2018
|
#49224
Thanks Jonathan,
Just came back to this game almost 2 months of studying later and I'm thankful to have made some major improvements because of you guys.

Get the most out of your LSAT Prep Plus subscription.

Analyze and track your performance with our Testing and Analytics Package.